Difference between revisions of "1965 AHSME Problems/Problem 3"
(→Solution) |
Mathfun1000 (talk | contribs) m (Explaining the solution clearly) |
||
Line 7: | Line 7: | ||
== Solution == | == Solution == | ||
− | + | Let us recall <math>PEMDAS</math>. We realize that we have to calculate the exponent first. <math>(-2)^{-2}=\frac{1}{(-2)^2}=\frac{1}{4}</math> When we substitute, we get <math>81^\frac{1}{4}=\sqrt[4]{81}=\boxed{\textbf{(C) }3}</math>. | |
+ | |||
+ | ~Mathfun1000 (Explaining clearly) |
Revision as of 08:05, 18 August 2021
Problem
The expression has the same value as:
Solution
Let us recall . We realize that we have to calculate the exponent first. When we substitute, we get .
~Mathfun1000 (Explaining clearly)